Cartridge Loading for a phono pre amp


Hello,

I have recently acquired a phono pre amp recommended by Michael Fremer.  It is “THE VINYL”, from QHW audio, Spain.  It got a great review.  I have a Benz Micro Glider rated at 1.1MV.  I have no idea how to set the dip switches for MC Load impedance for this cartridge. The options I have are as follows: 47K, 1K, 560R, 470R, 100R, and 47R.  I have a solid state amp and pre-amp, and also have a sub that I use, rarely.

Any advice would be most appreciated!!

judsauce

Showing 23 responses by holmz

holmz asked:

" Wouldn’t loading it down to 250 ohms from say 47k make it softer as the back EMF os gone that “would have been” stalling the motor? "

his answer was: " EMF?. No...." the same answer on other 2 subject treads different forums where was proved that it’s EMF:

"Yes, it really is back EMF- it’s calculated using Lentz’s law and is a consequence of Faraday’s Law of Induction and it occurs as a result of the change in current through the coil- that’s where the frequency dependent term comes from (the derivative). The term is subtracted from the voltage generated by the cartridge and in that way it acts to reduce the output voltage and hence the current, so there’s a degree of negative feedback. "

and in this thread you can find out the equations used. In this and the other threads your " man " "just " dead silence " against that EMF analysis the engeenering did it, not a single comment.

Thanks @rauliruegas

Are these the equations you are referencing?

 

 

My Sonic Lab Ultra Eminent BC: 0.6Ω output 0.29mV

 

Voltage Input: At 0.29 mV, the Wizard determined the optimal gain level at +70 dB of gain. This setting has audible hiss coming from the speakers at 30% volume without any record playing. On careful listening, the gain level was sufficient. Optimal loading was determined to be 180Ω by the Wizard.

 

Current Input: At 0.6 Ω, the Wizard determined the optimal gain level to be I/V + 20 dB, but based on listening test I preferred a much lower setting of I/V + 5 dB.

 

According to Ohm’s Law where I = V/R, Current = 0.29 / 0.6 = 483 Micro Amps, a very sufficient level of gain based on the low impedance of the cartridge.

 

Clearaudio Goldfinger Statement (GFS): 50Ω output 0.9mV

 

Voltage Input: At 0.9mV, the Wizard determined the optimal gain level to be 70dB of gain on the Voltage input. On listening test, I preferred +60 dB of gain, a much lower setting. Optimal loading was determined to be 330Ω by the Wizard.

 

Current Input: At 50Ω, the cartridge required IV+ 20 dB of gain.

 

According to Ohm’s Law where I = V/R, Current = 0.9/50 = 18 micro amps, the GFS’s high input impedance resulted in a very low level of gain regardless of the Goldfinger’s output of 0.9mV, which is relatively high for an MC cartridge.

If that is the case I still appear to need feeding to compute the back EMF.
Luckily you stated:

I think that all forum participants has a responsability and is not spread false information with no real evidence. That responsability comes by the fact that Agoners forums have different knowledge levels gentlemans and everyday comes rookies looking for advise looking for help so the higher knowledge level participants in the forums have all a higher responsability about that’s exactly that: give help and good proved advises.

 

Well… I am the rookie that needs some help.

Ralph stated that the cantilever gets stiffer as the impedance goes down,

you stated this most recently:

"Yes, it really is back EMF- it’s calculated using Lentz’s law and is a consequence of Faraday’s Law of Induction and it occurs as a result of the change in current through the coil- that’s where the frequency dependent term comes from (the derivative). The term is subtracted from the voltage generated by the cartridge and in that way it acts to reduce the output voltage and hence the current, so there’s a degree of negative feedback. "

and earlier this:
 

@holmz : " He never posted that loading to hard a cartridge will cause stiffness to the cartridge cantilever enough to mistracking. "

I was who posted that only to clarify that J.Carr never support those false statements by atmasphere. Tha’s all.

 

" to which answered " with false statements. Good for you if that is what you was looking for.

 

I paste all those information coming for true experts for we audiophiles can learn and don’t believe in what " some one " is spreading every where with out facts.

I would have assumed that the higher impedance was making it more difficult to push the current, and that the low impedance would allow it to flow easier and result in a lower voltage at the phono stage.

But Ralph says it is the other way, and another gentleman as well.
(Hence the statement I made about needing a real physicist.)

I would like to understand how the loading affects the voltage at the phone stage as well as the stiffness change at the cantilever/stylus.

Think this may be right:
 

If 100 ohms loading is too low and 470 ohms is too high, you can turn on both 470 ohms and 560 ohms to get approximately 250 ohms loading.

Certainly will be the closest.

holmz the compliance issue developed by changes in load impedance already was answered in this thread. Obviously not important for a normal audiophile because does not affects neither the LOMC cartridge frequency response

Yeah… Ralph fed me a while ago.

Hence I have no idea why you are still on me about being fed.
I am sated,

@noromance : " While mechanical impact does occur as a result of electrical load- there is some back emf necessarily generated by the signal current that affects the mechanical motion, but a quick back of the envelope calculation using Lenz’s law and the 10uH cartridge suggests a 2 orders of magnitude difference between the generated signal and the back EMF for a 100 ohm load at 20kHz- certainly not enough to cause tracking issue. "

Can you show the envelope and the equations?

 

Btw, @whistleraudio  posted: " we need a real physicist here. " and is rigth: he need it.

R

Who is whistler?

 

 

For acting like a psychic, may i ask where the LP is in the postal system?
And I should be picking up the ADC this weekend of next week.

I think I have the RIAA curve backwards though, so it would not help us at higher freqs with loading… exactly opposite.

@holmz Oh man, you're doing Raul's homework for him😄

If I cannot get fed, then at least I can do the cooking or the dishes ;)

Thanks @atmasphere .

I was under the impression that the current from the motor was the same with and without the load, so 47k would be a higher voltage across the resistance, and lower resistance would be less voltage.

Looks like I got it backwards.

@whistleraudio 

 

He never posted that loading to hard a cartridge will cause stiffness to the cartridge cantilever enough to mistracking.

It was my question of whether the cartridge was a current device, and whether the ease of pushing the current through the load affected the stiffness… to which @atmasphere answered.

I was not commenting on sound, just I was confused about back EMF type of affect upon the cartridge’s motor by having a high, versus low, ohm load.

Any change would be like the tail wagging the dog, and small compared to the masses ands mechanical compliance… and resonant frequency… of the arm/cart “system”.

@holmz : " He never posted that loading to hard a cartridge will cause stiffness to the cartridge cantilever enough to mistracking. "

It was I that wondered whether the loading affects the compliance in theory

I was who posted that only to clarify that J.Carr never support those false statements by atmasphere. Tha’s all.

" to which answered " with false statements. Good for you if that is what you was looking for.

It was not about mis tracking tests, it was about the motor affecting the stiffness in theory, whether or not it had impact on the sound was not my primary focus,

 

I paste all those information coming for true experts for we audiophiles can learn and don’t believe in what " some one " is spreading every where with out facts.

 

Unfortunatelly we are part of these corrupted AHEE. Such is life but people have the rigth to learn out of that AHEE.

what is AHEE?

 

Btw, who is: whistleraudio that you show in your last post.

a spell check on Raul I think.

 

 

Anyway in your last statement seems to me that in some way we are in agreement on that issue. Good.

R.

Maybe in agreement in that it doesn’t matter.

But maybe not in agreement as to whether the load being high or low would affect the compliance and which way it would do so if it did.

@judsauce The industry spec is 47K for all cartridges. When you load the cartridge at a lower resistance you are asking it to do more work (produce more power to drive that load). Even though its a tiny amount of power, it will make the cantilever stiffer and less able to trace high frequencies. It can and does affect the interaction between the arm and cartridge (effective mass and mechanical resonance).

Wouldn’t loading it down to 250 ohms from say 47k make it softer as the back EMF os gone that “would have been” stalling the motor?

Or do I have it backwards?

Thanks Ralph.

 

In this context, is there any relationship between cartridge bandwidth and cantilever stiffness? It seems that as the load-induced stiffening increases, the more reduced the bandwidth becomes.

It seems that way slightly, but there is an electrical component as well as mechanical.
And the stiffness may be more like a phase shift than an amplitude decrease??

we need a real physicist here.

@whistleraudio  aka @holmz  : I already made the " food " but you want that I " eat it " for you., no way.

You need to make your job.

I asked about compliance as a function of loading.
You provided frequency response as a function of loading.

It is like wanting an omelet, and receiving a half boiled egg, and claiming that the/a meal has been prepared.

We should probably just let it go.

In illustration using a very low output cartridge vs a very high output cartridge connected to the Current Input vs Voltage Input on the P1 

 

My Sonic Lab Ultra Eminent BC: 0.6Ω output 0.29mV 

 

Voltage Input: At 0.29 mV, the Wizard determined the optimal gain level at +70 dB of gain. This setting has audible hiss coming from the speakers at 30% volume without any record playing. On careful listening, the gain level was sufficient. Optimal loading was determined to be 180Ω by the Wizard. 

 

Current Input: At 0.6 Ω, the Wizard determined the optimal gain level to be I/V + 20 dB, but based on listening test I preferred a much lower setting of I/V + 5 dB. 

 

According to Ohm's Law where I = V/R, Current = 0.29 / 0.6 = 483 Micro Amps, a very sufficient level of gain based on the low impedance of the cartridge. 

  • Are you using ohms law with voltage to compute the output current?
  • I thought that the motors were generally current generating devices?
  • So is the impedance of the phono stage a DC reading?
  • And does and external loading become more like a V=I*R(total)

So no papers needed- just a grounding in basic physics (high school level), in particular how alternators and generators work. A 'Sheesh!' is in order here.

Ok I had HS physics and few more courses after HS.

 

Ohm's Law and the power formulas are not a figment of someone's imagination 😁

The cartridge is being asked to do just under 2 1/2 orders of magnitude more work with the example you provided:

47,000/100 = 470

-meaning 470X more current is drawn from the cartridge; because 47K is so high a resistance value relative to the impedance of the cartridge winding, the output voltage is essentially unaffected so the current increase also represents the wattage increase).

OK if the cart was a 0 ohms output impedance, then I think that the voltage would be linear, and inversely proportional to the impedance. 

As the cartirdges have an output impedance that is not zero, I am assuming that the produced current will not result in a linear relationship between voltage and loading, so that even with a 47k load the voltage will top out making the current either stall from flowing out of the cartridge, or ??

 

Anyone with an elementary school education can work out the math here. Perhaps, knowing that, you can tell me where that current is coming from??  If you can answer that, a bonus question: what is the consequence of that current flow?

The cartridge.

 

This is such simple math I don't see why a paper needs to be written about it, but maybe even though its basic, some people simply haven't thought it thru. Or didn't do so well in math.

I am hanging my head in shame, as I am assuming that the achieved current flow could be something like V/(R-cart +R-Load).
But I am not getting the simple ness of the math.

 

atmasphere @holmz : " " Your target for mechanical resonance is between 7-12Hz. The mechanical resonance is a product of the mass of the cartridge in the arm vs the compliance of the cantilever of the cartridge. Changing the load from 47K to 100 Ohms can easily get you outside of this target window- and that can cause tracking problems.. " "

 

Where out side in specific: 5hz? 15hz? 6hz? . All you have to do is show it.

 

 

R.

Ah you are talking about cartridge stiffness with respect to the arm/cart resonant frequency.
I was only wondering about the stillness changing.
And not wedding that to overly affecting the arm per se, but more as to whether the loading is affecting the cartridge stiffness, and maybe affecting the tracking of the cartridge.

So we are on two different targets, which are not intersecting.

I happy you have a wizard, I am not even sure what it is, but I like to at least have an intuitive feel for physics, if I cannot have a grasp on the equations.

I would want that grasp, irrespective of whether I had a machine that told me the answer.

 

This has been a fabulous technical lesson provided by one of the industries best. This has also been an exercise in personal credibility. Desperation vs confidence. Compliance vs torque. What a discussion; one for the ages

I am usually always confident 😎, so I have a start on it.

 

Again, you readed it in a wrong way because you or me can’t find out any " wizard " in my whole system.

 

What’s happening with you and with what you read here and every where?

 

R.

 

OK @rauliruegas what is this:

 

 

 

The P1 "Wizard"

 

As with all voltage amplification circuits, proper impedance loading is crucial to the resultant frequency response of the audio signal. The P1 provides a selection of five hundred resistance values from 20Ω all the way to 100kΩ in logarithmic increments; you’ll have steps of 1 ohm in the 20 Ohms range and the gaps between the values are increasing as you go upwards. The steps around 24k Ohms or above, are at 500 ohm increments. While most manufacturers will tell you to "go with your ear" while choosing the right loading, CH has developed an approach based on scientific measurements. The P1 is built in with an ingenious two part "Wizard" which will determine the optimal settings for GAIN and MC loading resistance automatically.

 

The P1 comes with a test LP designed to work with the P1’s internal distortion analyzer.

 

How reliable is the Wizard? I compared the results generated by the P1 versus the results from my own proprietary test LP and analog setup software currently under development. We arrived at the exact loading choice with a difference of only 10-20 ohms. Assuming we are relying on each other as the reliable benchmark, the results are close enough to be called scientifically verifiable! "

 

Swiss made too and expensive Dartzeel 18NS. MC load impedance up to 300ohms:

 

What's happening with you and with what you read here and every where?

I don’t know, I admit that I am having a hard time following along.

Since a cartridge is also a generator, asking it to make 400X more energy will affect it in a similar way. The only variable is that the cantilever has a springiness that affects the outcome of the equation. But its a simple fact that the coil itself will be 400X harder to move (this being between 47K load vs 100 Ohm load per Raul’s example).

If the voltage drops as the loading is applied i won’t be linear power.
And the output current also is likely non linear.
(I think)

 

Again, this principle is easy to demonstrate. Loudspeakers are moving coil, as they are mechanical transducers just as is a cartridge. If you try to move the woofer with your hand you’ll notice it takes a small amount of effort. Now put a short across the speaker terminals and see how easy it is to move the woofer- you’ll see its a lot stiffer. The same thing happens with a cartridge.

How does this concept affect the transimpedance phono amps?
I guess if they pretty much have 0 ohms input impedance then they would have the stiffest effect upon the cartridge? Rather than the softest?
(Compared to a traditional phono stage.)

Dear @holmz  : Did you follow the @imhififan  advise? , this is his advise:

 

" Perform a simple test with Hi-Fi News test record will give us the answer.."

Tests for resonance and tracking changing load impedance

I need to wait for it to arrive before I can use it..

@imhififan So sonically was the difference between 100 ohms and 47k readily apparent?

I put in a new stage a couple of weeks ago, and it arrived with a 1260 ohm loading and a SUT in-line. It was not a great sound... but I was excited.

So I pull it off the shelf, and opened it up to go to 47k for a MM cart with dip switched for no SUT.

And the sound came good.

@holmz 

I always prefer phono stage featured user select-able loading option, so the user can choose their preference.

And did you choose 100 ohms, 47 kph, or something in between?

As previously asked…:

 

So sonically was the difference between 100 ohms and 47k readily apparent?

@imhififan thanks mate!

On the subject of the tracking… the RIAA sort of works in our favour as the stylus does not have to move as far with the curve sloping down… maybe that lessens the cart getting stiffer for the high freqs? (It is but it hardly wriggles)

Hello, I am the poor guy that started this post with a simple question of pre-amp load settings.

Is anyone interested in my results when testing all of the settings on the pre-amp that you all recommended?

Si.

Please.